shoki

New Member
ارسال ها
637
لایک ها
128
امتیاز
0
#21
نمی دونم کجا رو اشتباه کردم ولی به نظر آسون میاد ... فرض کنید که a+bk بر p_1 و p_2 بخش پذیر باشه ... اونوقت اگه p_1 > p_2 داریم که :
یعنی کافیه که p_2 رو به اندازه ی دلخواه بزرگ کنیم(با باقیمانده چینی) ... پس از کران خبری نیس ... درستست؟؟

به جای اون p_n هم خیلی چیزای دیگه می شه گذاشت ...
 

mahanmath

New Member
ارسال ها
898
لایک ها
701
امتیاز
0
#22
mousavi گفت
8
فرض کنید
یک تصاعد حسابی از اعداد طبیعی باشد.برای هر n ,

بزرگ ترین عامل اول
میباشد.ثابت کنید دنباله
غیر کراندار می باشد.
فرض کنید k امین عضو تصاعد ak+b باشد .

حالا p رو یه عدد اول بگیرید که p به پیمانه a یک باشد و p از همه ی عوامل a+b بزرگتر باشه .

اعداد به شکل
عضو تصاعدند و جملات متناظر با اون ها تو دنباله با بزرگ کردن n به بی نهایت میل میکند .
 

mahanmath

New Member
ارسال ها
898
لایک ها
701
امتیاز
0
#23
2 سوال :

1)آیا برای هر n , تصاعدی غیر ثابت به طول n موجود است که دنباله ی مذکور صعودی (یا نزولی) باشد؟


2)ثابت باشد ؟

سوال 2 جوابش آره هست (
Tao-Green Theorem)
 

rezoos

New Member
ارسال ها
462
لایک ها
17
امتیاز
0
#24
پاسخ : ماراتن نظریه ی اعداد (سطح پیشرفته)

این سوال قسمت اولش خیلی بدیهیه یا من اشتباه میکنم?
 

MR.Amin

New Member
ارسال ها
594
لایک ها
202
امتیاز
0
#26
پاسخ : ماراتن نظریه ی اعداد (سطح پیشرفته)



ثابت کنید تنها a ای که به ازای هر n رابطه ی بالا مکعب کامل میشود 1 است!!!
 

mahdisaj

New Member
ارسال ها
183
لایک ها
3
امتیاز
0
#27
پاسخ : ماراتن نظریه ی اعداد (سطح پیشرفته)

عدد اول p مخالف 2 را در نظر می گیریم


a+1 ثابت است اما n متغیر است و می تواند 3k+1 یا 3k+2 یا 3k باشد. پس تناقض حاصل نشان می دهد که اگر مکعب کامل باشد باید توانی از دو باشد.
اگر a برابر 1 نباشد a>1 پس

(1)
اگر a به شکل 4k+1 باشد آنگاه برابری 1 برقرار نیست پس a به شکل 4k+3 است که برای n های روج برابری 1 برقرار نیست پس عدد مورد نظر توانی از 2 هم نیست که بزرگتر از 8 باشد .
پس فقط a می تواند 1 باشد
 

rezoos

New Member
ارسال ها
462
لایک ها
17
امتیاز
0
#28
پاسخ : ماراتن نظریه ی اعداد (سطح پیشرفته)

این لم 2 خط بود?
 

MR.Amin

New Member
ارسال ها
594
لایک ها
202
امتیاز
0
#29
پاسخ : ماراتن نظریه ی اعداد (سطح پیشرفته)

استفاده ای از اون بود!
 

hh1546

New Member
ارسال ها
76
لایک ها
65
امتیاز
0
#30
پاسخ : ماراتن نظریه ی اعداد (سطح پیشرفته)

البته بدون لم دو خط هم راحت اثبات میشه
 

MR.Amin

New Member
ارسال ها
594
لایک ها
202
امتیاز
0
#32
پاسخ : ماراتن نظریه ی اعداد (سطح پیشرفته)

نشان دهید برای هر n میتوان n عدد طبیعی پیدا کرد که مجموع آنه مربع کامل و حاصلضرب آنها مکعب کامل باشد!!!

(راهنمایی:از استقرا اگه خواستید استفاده کنید!!!)
 

mahdisaj

New Member
ارسال ها
183
لایک ها
3
امتیاز
0
#33

diba1993

New Member
ارسال ها
51
لایک ها
2
امتیاز
0
#34
پاسخ : ماراتن نظریه ی اعداد (سطح پیشرفته)

نشان دهید برای هر n میتوان n عدد طبیعی پیدا کرد که مجموع آنه مربع کامل و حاصلضرب آنها مکعب کامل باشد!!!

(راهنمایی:از استقرا اگه خواستید استفاده کنید!!!)
1^3,2^3,...,n^3
(يك بتوان 3 تا n به توان 3)
 
آخرین ویرایش توسط مدیر

hh1546

New Member
ارسال ها
76
لایک ها
65
امتیاز
0
#35
پاسخ : ماراتن نظریه ی اعداد (سطح پیشرفته)



#
 

Astronomer1

New Member
ارسال ها
32
لایک ها
8
امتیاز
0
#36
پاسخ : ماراتن نظریه ی اعداد (سطح پیشرفته)

تمام مقادیر ممکن بزرگ مقسوم علیه 2^{2n+1}+2^1}+{n+1 و 2^{2m+1}+2^{m+1}+1 را بیابید که در آن \left (2n+1, \right 2m+1) .

ظاهرا سوال در سطح سوال 1 مرحله 2 های متوسط/سخت+ به حساب می آد. ولی من به شخصه اینو سوال 2 مناسب تر می دونم برا مقایسه ی مرحله 2 ای با این رده. دلیلی هم پیش خودم برای این حرفم دارم.

من از آقای شریفی همین جا خواهش می کنم دوباره برامون سوال بنویسن و از دوستانی که تصمیم دارند جواب سوالی را بنویسند برای استدامت بیشتر از طرف نویسندگان سوال و برای قابل استفاده بودن سوالات برای همه بی زحمت لا اقل مختصر و مفید توضیحی راجع به حل بدهند. یه موضوع دیگه اینه لازم نیست اگه حوصله ندارید سوالو حلشو بنویسید بیاید و حتما پستی مثلا با محتوی این که این سوال با 'ویتا جامپینگ' یا چیزی مث اون show off کنین. منظورم اینه که یا ننویسین با برای همه بنویسین.

مر30
 

mahanmath

New Member
ارسال ها
898
لایک ها
701
امتیاز
0
#37
پاسخ : ماراتن نظریه ی اعداد (سطح پیشرفته)

تمام مقادیر ممکن بزرگ مقسوم علیه 2^{2n+1}+2^1}+{n+1 و 2^{2m+1}+2^{m+1}+1 را بیابید که در آن \left (2n+1, \right 2m+1) .

.

مر30
سلام ، میشه لطف کنی صورت سوال رو LaTeX کنی ؟ ممنون .
 

fereidoon

Active Member
ارسال ها
447
لایک ها
132
امتیاز
43
#38
پاسخ : ماراتن نظریه ی اعداد (سطح پیشرفته)

چی شد!!؟؟؟؟اقای شریفی تقاضای astronomer را قبول نمی کنید؟
 

fereidoon

Active Member
ارسال ها
447
لایک ها
132
امتیاز
43
#39
پاسخ : ماراتن نظریه ی اعداد (سطح پیشرفته)

ناسلامتی یه هفته مونده تا مرحله دو!!یعنی هیچکس نتونست اینو حل کنه؟؟!!
 

darrande

Well-Known Member
ارسال ها
592
لایک ها
811
امتیاز
93
#40
پاسخ : ماراتن نظریه ی اعداد (سطح پیشرفته)

یک سوال لم دار:
ثابت کنیدد برای هر n ،nعدد طبیعی متمایز وجود دارد طوری که حاصل جمع آنها مکعب کامل و حاصل ضرب آنها مربع کامل با شد.
توجه کنید این سوال با سوال مرحله دو سال 87 فرق دارد
 
بالا